Verbal questions from any Manhattan Prep GMAT Computer Adaptive Test. Topic subject should be the first few words of your question.
SixSigmaNinja
 
 

Advocates insist that health savings accounts

by SixSigmaNinja Fri Aug 31, 2007 7:41 pm

From Manhattan GMAT CAT 1

Advocates insist that health savings accounts are an efficient method to reduce medical expenses. However, widespread adoption of these accounts will soon undermine the public’s health. One reason for this is that most people will be reluctant to deplete their accounts to pay for regular preventive examinations, so that in many cases a serious illness will go undetected until it is far advanced. Another reason is that poor people, who will not be able to afford health savings accounts, will no longer receive vaccinations against infectious diseases.

The statements above, if true, most support which of the following?
A Wealthy individuals will not be affected negatively by health savings accounts.
B Private health insurance will no longer be available.
C Most diseases are detected during regular preventive examinations.
D Some people without health savings accounts are likely to contract infectious diseases.
E The causal relationship between an individual’s health and that person’s medical care has been adequately documented.


So my answer was E. It seems like there is a strong link between "The causal relationship between an individual’s health and that person’s medical care has been adequately documented" and "people will be reluctant to deplete their accounts to pay for regular preventive examinations, so that in many cases a serious illness will go undetected until it is far advanced"
Furthermore D seems to assume that the programs have been adopted (which is not stated or implied). With out that assumption the poor will be able to get vaccinations. What is up?


(D) CORRECT. The argument states that "poor people, who will not be able to afford health savings accounts, will no longer receive vaccinations". Based on this statement, it is reasonable to conclude that some people without health savings are likely to contract infectious diseases.

(E) The argument does not provide enough information to conclude that the causal relationship between an individual's health and that person’s medical care has been adequately documented. In fact, neither the link between medical care and health nor documentation of such a link is directly discussed.
StaceyKoprince
ManhattanGMAT Staff
 
Posts: 9350
Joined: Wed Oct 19, 2005 9:05 am
Location: Montreal
 

by StaceyKoprince Sat Sep 01, 2007 5:09 pm

First, pay attention to the structure of the question. It asks us to take the statements in the argument and use them to support an answer choice - not the other way around. In other words, this is a Draw a Conclusion question type. Conclusions must be completely supported by the given statements; we cannot make any assumptions or inferences in doing so.

In other words, if your reasoning is right, this statement: "people will be reluctant to deplete their accounts to pay for regular preventive examinations, so that in many cases a serious illness will go undetected until it is far advanced" would have to completely support this conclusion: "The causal relationship between an individual’s health and that person’s medical care has been adequately documented. " What does it mean to document a causal relationship? Perhaps studies have been done proving a link?

Now, in the real world, I certainly believe there have been many studies about the link between health and medical care. But do the premises presented in this argument address this topic at all, let alone strongly enough for me to draw a definitely-true conclusion?

For D, I agree with you somewhat and will ask our curriculum director to consider changing the language to say "if widely adopted" - I think that's where the issue lies.

However, I have seen things like this on the real test that I feel like I can argue with - yet they're on the real test, so the testwriters think they're fine. I think, if this were an OG question, they'd say that the opening sentence says HSAs "are an efficient method" - not that they will be, once they are implemented, but that they are right now. The later future language addresses not the mere existence of HSAs, but the "widespread adoption" of HSAs (sentence 2). Further, sentence 3 says that this widespread adoption "will soon undermine" - not the widespread adoption of HSAs would undermine, which implies that this adoption may or may not happen, but that it "will soon" undermine, meaning that it is actually happening.

Don't you just love the GMAT? Half the time, I feel like I'm on a debate team. :)
Stacey Koprince
Instructor
Director, Content & Curriculum
ManhattanPrep
SixSigmaNinja
 
 

Thanks Stacey

by SixSigmaNinja Sat Sep 01, 2007 6:55 pm

Stacey,

Thanks for your thoughtful reply. You are right. I chose E not because I was confident it was right but because the only other option was D (which left doubt).

What worries me about this question is the fact that E was an option that I had not ruled out. In light of your above posting D is clearly the most direct response. Another flaw in E is why would it need to be documented?

Thanks for your help. That's what makes Manhattan the best.
Captain
 
 

by Captain Mon Mar 31, 2008 5:07 am

I am not able to agree with the answer provided to this question.
The passage does not establish any connection between HSA and infectious diseases. In fact even the connection between vaccinations and diseases is not explicit but implied.

Further there is an issue with implementation of HSA as discussed in the post above.
I choose A after some internal debate. My reasoning was as follows.
- Mentioned that poor people will be at loss because of vaccines. Thus implied that the wealthy will not be affected.
- Some people will not want to deplete the account for preventive checks. But again people who can afford to pay from pocket even if account is depleted might not think twice before depleting their account.
I know that the second reasoning is vague but even then A seems the best option. I could not fit D into the answer.
Thanks
RonPurewal
Students
 
Posts: 19744
Joined: Tue Aug 14, 2007 8:23 am
 

by RonPurewal Wed Apr 02, 2008 4:31 am

hi cap'n

Captain Wrote:- Mentioned that poor people will be at loss because of vaccines. Thus implied that the wealthy will not be affected.


this sort of reasoning is cardinal sin. if i state a fact about group X, then, in general, i am allowed to conclude nothing about group Y.
what you're doing here is allowing your preconceived notions to get in the way of your critical thinking: you're taking the traditional polar opposition between the situation of the poor and that of the rich, and assuming that it applies to everything. there is no justification for doing so.

moreover, notice how general choice 'a' is: it asserts that wealthy people will not be adversely affected by the introduction of HSAs in any way. even if the vaccination issue is irrelevant, there may well be other issues that aren't. for instance, wealthy people might have to pay tens of thousands of dollars from an HSA for a surgery that would have cost then only a $50 co-pay on traditional health insurance. (note that it's not important for you to be able to concoct examples like this one; all that matters is that you realize that the statement is FAR too general to be even remotely supported by the passage.)

Captain Wrote:The passage does not establish any connection between HSA and infectious diseases.

ah, but it does.

remember to take the passage's conclusion into account in any evaluation of the significance of other statements.

the conclusion of this particular passage is: widespread adoption of [HSAs] will soon undermine the public’s health.
the final sentence of the passage, which is put forward as support for that conclusion, says that poor people, who will not be able to afford health savings accounts, will no longer receive vaccinations against infectious diseases.
read those two statements together a couple of times; they make a very strong connection between the two phenomena.
Captain
 
 

by Captain Thu Apr 03, 2008 4:13 am

Hi Ron,
Thanks for the post.
I can understand that any information about Group 1 does not give me any information on Group 2. But any information on Group 1 may tell me things about NOT (Group 1).
"The poor will always loose" => if you won you are not poor. (note that I have used NOT lost = won)
"Another reason is that poor people, who will not be able to afford health savings accounts, will no longer receive vaccinations against infectious diseases. "
=> If you recive vaccinations you are not poor. => if you recieve vaccines you are rich??? (assuming NOT poor = rich)... ahhh I see now. NOT poor = rich does not hold. (or does it? )
Well now the question is is NOT poor = rich?
RonPurewal
Students
 
Posts: 19744
Joined: Tue Aug 14, 2007 8:23 am
 

by RonPurewal Fri Apr 04, 2008 4:50 am

Captain Wrote:Hi Ron,
Thanks for the post.
I can understand that any information about Group 1 does not give me any information on Group 2. But any information on Group 1 may tell me things about NOT (Group 1).
"The poor will always loose" => if you won you are not poor. (note that I have used NOT lost = won)
"Another reason is that poor people, who will not be able to afford health savings accounts, will no longer receive vaccinations against infectious diseases. "
=> If you recive vaccinations you are not poor. => if you recieve vaccines you are rich??? (assuming NOT poor = rich)... ahhh I see now. NOT poor = rich does not hold. (or does it? )
Well now the question is is NOT poor = rich?


whoa man, you're shifting the goalposts here. the reasoning in this most recent post is solid.

BUT

the reasoning in the above post(s) translates, using this analogy, to
the poor will always lose
therefore
the rich will never lose
that's faulty logic.

go back and read your previous post again, and you'll realize that the reasoning you're putting forward in this newest post, although perfectly valid, is different from the logic of the previous post.

--

if you like formal terminology:

the logic in this most recent post is an example of the contrapositive:
if the statement 'if X then Y' is true, then the contrapositive 'if not Y, then not X' is also true.

the problem with the previous post is that you're trying to equate 'if X, then Y' to its inverse, which is 'if not X then not Y'. unfortunately, a statement and its inverse are two completely different assertions; they could be both true, both false, or one of each.

hope that helps
Captain
 
 

by Captain Mon Apr 07, 2008 5:14 am

Thanks,
You have a valid point.
rfernandez
Course Students
 
Posts: 381
Joined: Fri Apr 07, 2006 8:25 am
 

by rfernandez Fri May 02, 2008 3:05 pm

Good work, all!
chandran.sharat
Students
 
Posts: 4
Joined: Mon Jul 13, 2009 7:33 pm
 

Re: Advocates insist that health savings accounts

by chandran.sharat Fri Nov 20, 2009 5:21 am

I'm sorry but I dont get it.

"...so that in many cases a serious illness will go undetected until it is far advanced. "

Doesnt this mean that if there were 100 ppl, the disease would go undetected in say 51 of them? This, I believe, helps us conclude that most diseases are detected during preventive examination (C)

I got confused between C and D and chose one at random.

Thank You
hrishikesh.karve
Students
 
Posts: 1
Joined: Thu Dec 24, 2009 1:40 am
 

Re:

by hrishikesh.karve Mon Jan 04, 2010 3:25 am

StaceyKoprince Wrote:First, pay attention to the structure of the question. It asks us to take the statements in the argument and use them to support an answer choice - not the other way around. In other words, this is a Draw a Conclusion question type. Conclusions must be completely supported by the given statements; we cannot make any assumptions or inferences in doing so.

In other words, if your reasoning is right, this statement: "people will be reluctant to deplete their accounts to pay for regular preventive examinations, so that in many cases a serious illness will go undetected until it is far advanced" would have to completely support this conclusion: "The causal relationship between an individual’s health and that person’s medical care has been adequately documented. " What does it mean to document a causal relationship? Perhaps studies have been done proving a link?

Now, in the real world, I certainly believe there have been many studies about the link between health and medical care. But do the premises presented in this argument address this topic at all, let alone strongly enough for me to draw a definitely-true conclusion?

For D, I agree with you somewhat and will ask our curriculum director to consider changing the language to say "if widely adopted" - I think that's where the issue lies.

However, I have seen things like this on the real test that I feel like I can argue with - yet they're on the real test, so the testwriters think they're fine. I think, if this were an OG question, they'd say that the opening sentence says HSAs "are an efficient method" - not that they will be, once they are implemented, but that they are right now. The later future language addresses not the mere existence of HSAs, but the "widespread adoption" of HSAs (sentence 2). Further, sentence 3 says that this widespread adoption "will soon undermine" - not the widespread adoption of HSAs would undermine, which implies that this adoption may or may not happen, but that it "will soon" undermine, meaning that it is actually happening.

Don't you just love the GMAT? Half the time, I feel like I'm on a debate team. :)



I agree to what you have said Stacey.
But, still if we take a close look on the main idea what "C" is saying it does not look incorrect.

As per the question, below is the one reason that we should consider to be true.

"One reason for this is that most people will be reluctant to deplete their accounts to pay for regular preventive examinations, so that in many cases a serious illness will go undetected until it is far advanced"

Above thing explains that if we do not detect disease at a preliminary stage, it can cause serious illness later on(or in advance stages) and would prove more dangerous. Hence, if I say that "preliminary treatment is the most important as most or maximum number of diseases can be diagnosed through it.", it would help to support the above reason that many or most of the people will get disease. Option C states the same thing.

Also, our ultimate goal is -- to prove that more and more people will get disease if there is a widespread adoption of these accounts.

Whereas option D states that some people without HSA are likely to contract infectious diseases, this does not mean that the many people with HSA will get disease. In fact, option D makes an assumption.

For example, If I say - some students without degrees are likely to remain jobless.

But that does not mean that many people with degrees/without degrees will get job or will remain jobless.

Stacey, can you please give more explanation for why option C is incorrect?

The reason that you had explained above does not consider this point, that most people or many people getting a disease will fulfill our ultimate goal to undermine public heath, than using SOME as in D and making an converse assumption about many.
_________________
Hrishi

"As you sow, so shall you reap"
Ben Ku
ManhattanGMAT Staff
 
Posts: 817
Joined: Sat Nov 03, 2007 7:49 pm
 

Re: Advocates insist that health savings accounts

by Ben Ku Sun Apr 04, 2010 1:16 am

chandran.sharat Wrote:I'm sorry but I dont get it.

"...so that in many cases a serious illness will go undetected until it is far advanced. "

Doesnt this mean that if there were 100 ppl, the disease would go undetected in say 51 of them? This, I believe, helps us conclude that most diseases are detected during preventive examination (C)

I got confused between C and D and chose one at random.

Thank You


In the statement from the argument, "in many cases a serious illness will go undetected ..." points to the result when many people don't go in for preventative exams. It does not give us any information about what WILL happen if people have regular preventative exams. We would suspect that more illness would be detected, but we don't know if "most of them" would be detected.

However, answer choice (C) claims that most will be detected by regular preventative exams, which goes too far and is unsupported by the argument. The problem with using the statement in the argument you quote to justify (C) is that you're comparing two completely separate situations: when people go NOT go in for prevent. exams vs. when people DO go in for these exams.

Hope that helps.

The problem with using
Ben Ku
Instructor
ManhattanGMAT
Ben Ku
ManhattanGMAT Staff
 
Posts: 817
Joined: Sat Nov 03, 2007 7:49 pm
 

Re: Re:

by Ben Ku Sun Apr 04, 2010 1:29 am

hrishikesh.karve Wrote:I agree to what you have said Stacey.
But, still if we take a close look on the main idea what "C" is saying it does not look incorrect.

As per the question, below is the one reason that we should consider to be true.

"One reason for this is that most people will be reluctant to deplete their accounts to pay for regular preventive examinations, so that in many cases a serious illness will go undetected until it is far advanced"

Above thing explains that if we do not detect disease at a preliminary stage, it can cause serious illness later on(or in advance stages) and would prove more dangerous. Hence, if I say that "preliminary treatment is the most important as most or maximum number of diseases can be diagnosed through it.", it would help to support the above reason that many or most of the people will get disease. Option C states the same thing.


See my comment above. Mainly the word "most" goes too far. The argument indicates that more diseases will be detected, but it does not provide information about how many.

Whereas option D states that some people without HSA are likely to contract infectious diseases, this does not mean that the many people with HSA will get disease. In fact, option D makes an assumption.

For example, If I say - some students without degrees are likely to remain jobless.

But that does not mean that many people with degrees/without degrees will get job or will remain jobless.

The reason that you had explained above does not consider this point, that most people or many people getting a disease will fulfill our ultimate goal to undermine public heath, than using SOME as in D and making an converse assumption about many.


The proof for choice D is in the last sentence "poor people, who will not be able to afford health savings accounts, will no longer receive vaccinations against infectious diseases. " It's true that it makes an assumption that if they don't receive vaccinations, then they will contract infectious diseases. However, this is not much a stretch; why else would one be vaccinated? Because it prevents the disease that otherwise one might contract.

Your thought that we cannot assume that the converse of a statement is keen. However, it only applies to a conditional statement (If X, then Y). Here, the sentence is only a claim.
Ben Ku
Instructor
ManhattanGMAT
jp.jprasanna
Students
 
Posts: 200
Joined: Thu Nov 03, 2011 3:48 am
 

Re: Advocates insist that health savings accounts

by jp.jprasanna Sat Jun 30, 2012 3:58 am

Ben Ku Wrote:
chandran.sharat Wrote:I'm sorry but I dont get it.

"...so that in many cases a serious illness will go undetected until it is far advanced. "

Doesnt this mean that if there were 100 ppl, the disease would go undetected in say 51 of them? This, I believe, helps us conclude that most diseases are detected during preventive examination (C)

I got confused between C and D and chose one at random.

Thank You


In the statement from the argument, "in many cases a serious illness will go undetected ..." points to the result when many people don't go in for preventative exams. It does not give us any information about what WILL happen if people have regular preventative exams. We would suspect that more illness would be detected, but we don't know if "most of them" would be detected.

However, answer choice (C) claims that most will be detected by regular preventative exams, which goes too far and is unsupported by the argument. The problem with using the statement in the argument you quote to justify (C) is that you're comparing two completely separate situations: when people go NOT go in for prevent. exams vs. when people DO go in for these exams.

Hope that helps.

The problem with using


Advocates insist that health savings accounts are an efficient method to reduce medical expenses. However, widespread adoption of these accounts will soon undermine the public’s health. One reason for this is that most people will be reluctant to deplete their accounts to pay for regular preventive examinations, so that in many cases a serious illness will go undetected until it is far advanced. Another reason is that poor people, who will not be able to afford health savings accounts, will no longer receive vaccinations against infectious diseases.

The statements above, if true, most support which of the following?
C Most diseases are detected during regular preventive examinations.

Dear Ben / Ron - I didn't even jump into the modifier analysis as I thought there was scope shift in the answer choice....

Passage reads - so that in many cases a serious illness will go undetected

Option B says - Most diseases are detected during regular preventive examinations

SERIOUS ILLNESS and DISEASES are the not the same aren't they?

Cheers
jnelson0612
ManhattanGMAT Staff
 
Posts: 2664
Joined: Fri Feb 05, 2010 10:57 am
 

Re: Advocates insist that health savings accounts

by jnelson0612 Sat Jun 30, 2012 10:33 pm

jp.jprasanna Wrote:
Ben Ku Wrote:
chandran.sharat Wrote:I'm sorry but I dont get it.

"...so that in many cases a serious illness will go undetected until it is far advanced. "

Doesnt this mean that if there were 100 ppl, the disease would go undetected in say 51 of them? This, I believe, helps us conclude that most diseases are detected during preventive examination (C)

I got confused between C and D and chose one at random.

Thank You


In the statement from the argument, "in many cases a serious illness will go undetected ..." points to the result when many people don't go in for preventative exams. It does not give us any information about what WILL happen if people have regular preventative exams. We would suspect that more illness would be detected, but we don't know if "most of them" would be detected.

However, answer choice (C) claims that most will be detected by regular preventative exams, which goes too far and is unsupported by the argument. The problem with using the statement in the argument you quote to justify (C) is that you're comparing two completely separate situations: when people go NOT go in for prevent. exams vs. when people DO go in for these exams.

Hope that helps.

The problem with using


Advocates insist that health savings accounts are an efficient method to reduce medical expenses. However, widespread adoption of these accounts will soon undermine the public’s health. One reason for this is that most people will be reluctant to deplete their accounts to pay for regular preventive examinations, so that in many cases a serious illness will go undetected until it is far advanced. Another reason is that poor people, who will not be able to afford health savings accounts, will no longer receive vaccinations against infectious diseases.

The statements above, if true, most support which of the following?
C Most diseases are detected during regular preventive examinations.

Dear Ben / Ron - I didn't even jump into the modifier analysis as I thought there was scope shift in the answer choice....

Passage reads - so that in many cases a serious illness will go undetected

Option B says - Most diseases are detected during regular preventive examinations

SERIOUS ILLNESS and DISEASES are the not the same aren't they?

Cheers


Since the previous explanations aren't helping, let me approach this from a different angle. This question type is draw a conclusion; the correct answer MUST be true based on the content of the passage. With this type of question you have to be very, very careful with the wording. The passage actually says:
"in many cases a serious illness will go undetected until it is far advanced".

How many is many? Many is certainly more than one. Is it half or more than half? Not necessarily.

C says "Most diseases are detected during regular preventative examinations."

Because of this difference, I can't conclude "most" when the passage only says "many".

*Most* is a more specific word, which means more than half. Consider these different sentences:

Many of my friends have been to Europe. (more than one, and could be the majority, but we don't know exactly how many)
Most of my friends have been to Europe. (definitely more than half have been to Europe)

Does this help? Notice how the correct answer, D, uses lots of wishy-washy, moderate words so you don't run into this problem. D is really an ideal answer for this type of question because it is so hard to argue with.
Jamie Nelson
ManhattanGMAT Instructor